Expected value and joint probability density function

Click For Summary
The joint probability density function for two uniformly distributed random variables (X,Y) on a circle of radius a is defined as f(x,y) = 1/(πa²) for x² + y² ≤ a², and 0 otherwise. The expected value of X is initially proposed as E(X) = ∫∫(x/(πa²)) dx dy, but the limits of integration must be adjusted to reflect the circle's boundaries. Correct limits for the integral should be from -a to a for y, and from -√(a² - y²) to √(a² - y²) for x. It is suggested that using polar coordinates simplifies the integration, with appropriate limits for r and θ to cover the entire circle.
kasse
Messages
383
Reaction score
1
Assume that two random variables (X,Y) are uniformly distributed on a circle with radius a. Then the joint probability density function is

f(x,y) = \frac{1}{\pi a^2}, x^2 + y^2 <= a^2
f(x,y) = 0, otherwise

Find the expected value of X.


E(X) = \int^{\infty}_{- \infty}\int^{\infty}_{- \infty}\frac{x}{\pi a^2} dxdy

Is this correct so far? What are the limits of the integral supposed to be?
 
Physics news on Phys.org
Since your pdf is nonzero only on the circle, I think this is what you want:
\int^{a}_{-a}\int^{\sqrt{a^2 - y^2}}_{- \sqrt{a^2 - y^2}}\frac{x}{\pi a^2} dxdy
 
I think this integral would be easier in polar coordinates.
 
kasse said:
Assume that two random variables (X,Y) are uniformly distributed on a circle with radius a. Then the joint probability density function is

f(x,y) = \frac{1}{\pi a^2}, x^2 + y^2 <= a^2
f(x,y) = 0, otherwise

Find the expected value of X.


E(X) = \int^{\infty}_{- \infty}\int^{\infty}_{- \infty}\frac{x}{\pi a^2} dxdy

Is this correct so far? What are the limits of the integral supposed to be?
No that is not correct because the limits of integration cannot be from -\infty to \infty. The probability outside the circle of radius a is 0, not 1/(\pi a^2).

In polar coordinates the integral would be
\frac{1}{\pi a^2}\int_r\int_\theta r cos(\theta) (r drd\theta)
and you want to cover the circle of radius a. What are the limits of integration for that?
 
Thanks!
 
Question: A clock's minute hand has length 4 and its hour hand has length 3. What is the distance between the tips at the moment when it is increasing most rapidly?(Putnam Exam Question) Answer: Making assumption that both the hands moves at constant angular velocities, the answer is ## \sqrt{7} .## But don't you think this assumption is somewhat doubtful and wrong?

Similar threads

Replies
9
Views
2K
  • · Replies 40 ·
2
Replies
40
Views
4K
  • · Replies 19 ·
Replies
19
Views
4K
  • · Replies 11 ·
Replies
11
Views
2K
  • · Replies 3 ·
Replies
3
Views
1K
  • · Replies 2 ·
Replies
2
Views
1K
  • · Replies 1 ·
Replies
1
Views
2K
Replies
2
Views
2K
  • · Replies 6 ·
Replies
6
Views
2K
  • · Replies 14 ·
Replies
14
Views
2K